Can someone help I’ll give 10 points

Can Someone Help Ill Give 10 Points

Answers

Answer 1

Answer:

Hey Sweetheart!

z=4

4x4x4= 64

Hope this Helps!!!


Related Questions

Noah was at home. He got on his bike and rode to his friends

Answers

Answer:

what's your exact question

Answer:

can u pls type the full question

what is the probability that the first question she gets right is question number 4? group of answer choices

Answers

The probability that the first question she gets right is question number 4, is 0.1054.

Number of options there are for a single query = 4

P(guessing correct answer for a single question) = 1/4

P(guessing correct answer for a single question) = 0.25

Probability of getting correct answer P(correct) = 0.25

Probability of getting wrong answer P(wrong) = 1 - Probability of getting correct answer

Probability of getting wrong answer P(wrong) = 1 - 0.25

Probability of getting wrong answer P(wrong) = 0.75

So, the probability that the first question she gets right is question number 4 = Probability of getting 1st question wrong × Probability of getting 2nd question wrong × Probability of getting 3rd question wrong × Probability of getting 4th question right

The probability that the first question she gets right is question number 4 = 0.75 × 0.75 × 0.75 × 0.25

The probability that the first question she gets right is question number 4 = 0.1055

To learn more about probability link is here

brainly.com/question/31120123

#SPJ4

The complete question is:

In a multiple choice exam, there are 5 questions and 4 choices for each question (a, b, c, d). Nancy has not studied for the exam at all and decides to randomly guess the answers. (Round your answers to four decimal places.)

What is the probability that the first question she gets right is question number 4?

9 km
7 km
3 km
3 km
3 km
2 km
8 km
9 km
3 km
7 km

Answers

Answer: what do you mean? I need more info-

Step-by-step explanation:

I can answer it with more info :)

which value of n makes the equation true
-[tex]\frac{1}{2}n=-8[/tex]

Answers

Answer:

16

Step-by-step explanation:

When we divide the entire equation by 1/-2 to get rid of the coefficient of n we get n = 16.

PLEASE HELP An elevator goes down 3 floors, up 5 floors, and finally down 7 floors. Which expression represents the total number of floors the elevator traveled?

*
A | -3 | + | 5 | + | -7 |
B | -3 | - | 5 | + | -7 |
C ( -3 ) + 5 + ( -7 )
D 3 - 5 + 7

Answers

Answer:

B is the answer

Step-by-step explanation:

the radius of a circle is changing at .5 cm/sec. find the rate of change of the area when the radius is 4 cm.

Answers

The rate of change of the area of a circle when the radius is 4 cm is 4π cm2/sec.

This can be calculated using the formula for the area of a circle (A = πr2) and the chain rule for derivatives. The chain rule states that when the radius (r) changes, the area of a circle (A) is equal to 2πr times the rate of change of the radius (dr/dt).

Therefore, the rate of change of the area of a circle when the radius is 4 cm is equal to 2π(4 cm) × (0.5 cm/sec) = 4π cm2/sec.

Note that if the rate of change of the radius were different, the rate of change of the area would also be different. This formula can be used to calculate the rate of change of the area of a circle at any given radius, as long as the rate of change of the radius is known.

For more such questions on Rate of change of area.

https://brainly.com/question/30435679#

#SPJ11

If g(x) = 1 – 2x + 3x2, find the average rate of change of the function as x varies from 2 to 5

Answers

The average rate of change of the function as x varies from 2 to 5 is 22.

Given function: g(x) = 1 – 2x + [tex]3x^2[/tex]

To find the average rate of change of the function as x varies from 2 to 5.

Solution: We are given a function: g(x) = 1 – 2x + [tex]3x^2[/tex]

The average rate of change of the function as x varies from a to b is given by:

Average rate of change = f(b) - f(a) / b - a

Let a = 2 and b = 5

We have to find the average rate of change of g(x) as x varies from 2 to 5.

So, the average rate of change of g(x) is given by:

Average rate of change = g(5) - g(2) / 5 - 2

= [1 - 2(5) + 3([tex]5^2[/tex])] - [1 - 2(2) + 3([tex]2^2[/tex])] / 3

= [1 - 10 + 75] - [1 - 4 + 12] / 3= 66 / 3= 22

Therefore, the average rate of change of the function as x varies from 2 to 5 is 22.

An average rate of change is the amount that the function changes on average over a specified interval.

The formula for average rate of change is given as the change in the function value divided by the change in x value for two distinct points on the function.

For similar question on function.

https://brainly.com/question/22340031

#SPJ11

To make cleaning easier, a rectangular horse trough will be lined with plastic. The trough is 40 inches long, 14 inches wide, and 24 inches deep. How many square inches of plastic are needed to line the trough? Count only the trough's five faces. A net containing 5 rectangles. Two rectangles have length of 40 inches and width of 14 inches. Two rectangles have length of 14 inches and width of 24 inches. One rectangle has length of 40 inches and width of 24 inches.

Answers

Using the area formula for the rectangle, we can find that 2752 in² of plastic is needed to line the trough.

Define area?

To determine the area a rectangle occupies within its perimeter, apply the formula for calculating a rectangle's area. Multiplying the length by the width yields the area of a rectangle (breadth).

As a result, the area of a rectangle with the length and breadth l and w, respectively, is calculated as follows. L × W = the rectangle's area. Hence, the area of a rectangle is equal to (length width).

Now in the given question,

We have 5 faces of the cuboid.

Now to find the total area of the required space we have to find the area of all the rectangles.

Area of rectangle with dimensions, l = 40inches and b = 14 inches.

Area = l × b

= 40 × 14

= 560in²

Now there are 2 rectangles with the same dimensions, so the total area = 560 + 560 = 1120in².

Now area of rectangles with dimensions, l = 14 inches and b = 24 inches.

Area = l × b

= 14 × 24

= 336in².

There are 2 rectangles with the same dimensions, so area = 336 + 336 = 672in².

Area of the final rectangle = l × b

= 40 × 24

= 960in².

So, the total required area = 1120 + 672 + 960 = 2752in².

To know more about area of rectangles, visit:

https://brainly.com/question/16309520

#SPJ1

8x 2 + [ 3x3-8] = with explanation pls and its due in six minutes

Answers

Answer: 16x+3x^3−8

Please mark me brainliest :)

Determine whether segment lengths form a triangle. If so, classify the triangle as acute, right or obtuse.

1. 10, 7, sqrt(658)

Answers

Answer:

it is a triangle bc it has angles of points

Step-by-step explanation:

which of the following conditions must be met to conduct a two-proportion significance test? the populations are independent. the probabilities of success multiplied by the sample sizes are greater than or equal to 10 and the probabilities of failure multiplied by the sample sizes are greater than or equal to 10 for each population. the sample sizes are greater than 30.

Answers

The following conditions must be met to conduct a two-proportion significance test:

the populations are independent, the probabilities of success multiplied by the sample sizes are greater than or equal to 10 and the probabilities of failure multiplied by the sample sizes are greater than or equal to 10 for each population, and the sample sizes are greater than 30.

The two-proportion significance test is a hypothesis test that compares the proportions of two independent populations.

To conduct the two-proportion significance test, the following conditions must be met:

Populations must be independent.

Sample sizes are greater than 30.

The probabilities of success multiplied by the sample sizes are greater than or equal to 10 and the probabilities of failure multiplied by the sample sizes are greater than or equal to 10 for each population.

The sample size should be large enough so that the sampling distribution of the sample proportion is nearly normal. The sample sizes should be large enough so that the central limit theorem can be applied.

In short, to conduct a two-proportion significance test, the populations must be independent, the probabilities of success multiplied by the sample sizes are greater than or equal to 10 and the probabilities of failure multiplied by the sample sizes are greater than or equal to 10 for each population, and the sample sizes are greater than 30.


To know more about two-proportion significance test refer here:

https://brainly.com/question/30255520#

#SPJ11

Help me with these please!!

Answers

The angle ABD is 35 degrees, AC is 20 units long, and AB is 29 units long.

What in mathematics is an angle?

An angle is created by combining two rays (half-lines) that have a common terminal. The angle's vertex is the latter, while the rays are alternately referred to as the angle's legs and its arms.

Triangle ABD's angle ABC is one of its outside angles, making it equal to the sum of the opposing interior angles.

Angle ABC = Angle ABD + Angle ACD

replacing the specified values:

110° = Angle ABD + 75°

Simplifying:

Angle ABD = 110° - 75°

Angle ABD = 35°

Due of their shared angles, the two triangles are comparable. This fact can be used to establish a ratio between the corresponding sides:

AC / CD = AB / BD

replacing the specified values:

AC / 10 = 16 / 8

Simplifying:

AC = 20

To know more about angle visit:-

brainly.com/question/28451077

#SPJ1

Complete Question:

Find the angle ∠ABD jn the given figure

a children's liquid medicine contains 100 mg of the active ingredient in 5 ml . if a child should receive 300 mg of the active ingredient, how many milliliters of the medicine should the child be given? for the purposes of this question, assume that these numbers are exact.

Answers

The child should be given 15 ml of the medicine to receive 300 mg of the active ingredient.

The given problem requires us to determine the number of milliliters of a liquid medicine that a child should receive in order to obtain a specific dosage of the active ingredient. We are given that the medicine contains 100 mg of the active ingredient in 5 ml.

The child needs to receive 300 mg of the active ingredient, and there are 100 mg of the active ingredient in 5 ml of the medicine. Therefore, the child should be given:

[tex]\frac{300 mg}{100mg/5ml} = \frac{300\text{ mg} \times 5\text{ ml}}{100\text{ mg}} = 15\text{ ml}$$[/tex]

So the child should be given 15 ml of the medicine to receive 300 mg of the active ingredient.

Learn more about Liquid Measurements:

https://brainly.com/question/7744911
#SPJ4

suppose there is $600 in the account with an annual interest rate of 4%. after how many years will the amount triple?

Answers

it will take approximately 22.56 years for the amount to triple.

The given information for this problem is that there is an initial investment of $600 in an account with an annual interest rate of 4%. The task is to determine after how many years the amount will triple.Using the compound interest formula, we can find the amount in the account after t years:A = P(1 + r/n)nt Where,A = final amount in the account, P = initial amount in the account r = annual interest rate ,n = number of times the interest is compounded per year ,t = time in years.

From the problem statement, we know that the initial amount, P, is $600 and the annual interest rate, r, is 4%. Let's assume that the interest is compounded annually, i.e., n = 1.Substituting these values in the formula, we get:A = $600(1 + 0.04/1)1t Simplifying this expression,A = $600(1.04)t.

Taking the ratio of the final amount to the initial amount, we get: 3P = $600 × 3 = $1800. Therefore,A/P = 3 = (1.04)t.Dividing both sides by P, we get:3 = (1.04)t ln(3) = ln(1.04)t. Using the logarithmic property, we can bring down the exponent to the front:ln(3) / ln(1.04) = t Using a calculator, we get ≈ 22.56. Therefore, it will take approximately 22.56 years for the amount to triple.

To know more about compound interest, click here:

https://brainly.com/question/22621039

#SPJ11

GIVING BRAINLIEST FOR THE CORRECT ANSWER (i need a proof that what you’re saying is right bc ppl are giving me the wrong answers)

Answers

Answer:

x [tex]\geq[/tex]2

Step-by-step explanation:

Since the arrow is pointing to the right, we know that it is greater than two. We also know that it could be equal to 2 because the dot is filled in on the number line. So, the answer is x is greater than or equal to 2.

Help please, Which value of x satisfies the equation 7/3(x+9/28)=20

Answers

Answer:

Step-by-step explanation:

[tex]\frac{7}{3} (x+\frac{9}{28} )=20[/tex]    

[tex]7 (x+\frac{9}{28} )=60[/tex]     (multiplied both sides by 3)

[tex]x+\frac{9}{28} =\frac{60}{7}[/tex]          (divided both sides by 7)

[tex]x=\frac{60}{7}-\frac{9}{28}=\frac{240}{28}-\frac{9}{28}=\frac{231}{28} =8.25[/tex]     (subtracted [tex]\frac{9}{28}[/tex] both sides and solved)

if the matrix product a1b is known, how could you calculate b1a without necessarily knowing what a and b are?

Answers

We can calculate its product by taking the dot product of each row of B1A and each column of A1B. In this way, we can calculate B1A without knowing the values of A and B.

The matrix product of two matrices, A and B, is defined as the matrix C, where C = AB. To calculate the product of two matrices, we must take the dot product of each row of A and each column of B. If we are given a matrix product A1B, then we can calculate B1A without necessarily knowing what A and B are.

To do so, we must first invert the matrix A1B. We can do this by solving a system of equations. We can set up this system of equations by treating the entries of A1B as the coefficients in a system of equations, and solving for the entries of B1A. Once we have found the inverse, we can calculate the matrix B1A.

Finally, once we have the matrix B1A, we can calculate its product by taking the dot product of each row of B1A and each column of A1B. In this way, we can calculate B1A without knowing the values of A and B.

See more about matrix at: https://brainly.com/question/14414517

#SPJ11

a box contains 8 red balls and 8 blue balls, and 4 balls are taken at random without replacement. what is the probability that 2 red balls and 2 blue balls are taken?

Answers

The probability that 2 red balls and 2 blue balls are taken from the box is 3/7. This can be expressed mathematically as [tex](8C2 * 8C2) / (16C4) = 3/7[/tex].

To better understand this probability, let's look at an example. Say there are 8 red balls and 8 blue balls in the box. This can be represented as:

R = 8, B = 8

We want to determine the probability of taking 2 red balls and 2 blue balls out of the box. To do this, we need to calculate the total number of ways of selecting 4 balls from the box (16 balls in total) and then calculate the total number of ways of selecting 2 red and 2 blue balls out of the box.

The total number of ways of selecting 4 balls from the box can be expressed as (16C4). This is calculated by dividing the number of ways of selecting 4 balls out of 16 (16!) by the number of ways of arranging those 4 balls in any order (4!):

[tex](16C4) = 16! / 4! = 1820[/tex]

The total number of ways of selecting 2 red and 2 blue balls out of the box can be expressed as (8C2 * 8C2). This is calculated by multiplying the number of ways of selecting 2 red balls out of 8 (8C2) by the number of ways of selecting 2 blue balls out of 8 (8C2):

[tex](8C2 * 8C2) = 8C2 * 8C2 = 28[/tex]

The probability of taking 2 red balls and 2 blue balls out of the box is then the ratio of the number of ways of selecting 2 red and 2 blue balls out of the box (28) to the total number of ways of selecting 4 balls from the box (1820):

P(2 red balls, 2 blue balls) = 28 / 1820 = 3/7

In conclusion, the probability of taking 2 red balls and 2 blue balls out of a box containing 8 red balls and 8 blue balls is 3/7.

See more abut probability at: https://brainly.com/question/24756209

#SPJ11

44.0183 rounded to the nearest thousands

Answers

44.0180 since 8 cannot be rounded because of 3.

Angle PQR is isosocles with PQ=PR= 7. 5cm and QR = 9cm. The height PS from P to QR,is 6cm. Find the area of Angle PQR. What will be the height from R to PQ that is RT

Answers

The height RT from R to PQ is approximately 3.16 cm.

To find the area of triangle PQR, we can use the formula:

Area = 1/2 * base * height

Since PQR is isosceles with PQ = PR, the base is PQ or PR. We can choose PQ as the base. Then the height is PS.

Area of PQR = 1/2 * PQ * PS

Since PQ = PR = 7.5 cm and PS = 6 cm, we can substitute these values into the formula and simplify:

[tex]Area of PQR = 1/2 * 7.5 cm * 6 cm[/tex]

[tex]Area of PQR = 22.5 cm^2[/tex]

Therefore, the area of triangle PQR is [tex]22.5 cm^2[/tex].

To find the height RT from R to PQ, we can use the Pythagorean theorem.

Let's draw a perpendicular line from R to PQ, intersecting at T. Then we have a right triangle PRT with hypotenuse PR and legs PT and RT.

Since PQR is isosceles, we can also see that angle PQR is equal to angle PRQ. Therefore, angles PQR and PRQ are equal and each is approximately 69.3 degrees (using inverse cosine function).

Using the sine function, we can find the length of PT:

sin(69.3) = PT / 7.5

PT = 7.5 * sin(69.3)

PT ≈ 6.93 cm

Using the Pythagorean theorem, we can find the length of RT:

[tex]RT^2 + PT^2 = PR^2[/tex]

[tex]RT^2 = PR^2 - PT^2[/tex]

[tex]RT^2 = 7.5^2 - 6.93^2[/tex]

RT ≈ 3.16 cm

Learn more about area of triangle here https://brainly.com/question/19305981

#SPJ4

if the odds on a bet are 16:1 against, what is the probability of winning? express your answer as a fraction.

Answers

The probability of winning is 1/17, which can also be expressed as a decimal (approximately 0.059) or as a percentage (approximately 5.9%).

The odds on a bet represent the ratio of the probability of winning to the probability of losing. In this case, the odds are 16:1 against winning, which means that the probability of winning is 1 out of 16.

To express this probability as a fraction, we can use the formula:

Probability of winning = 1 / (odds + 1)

Plugging in the given odds, we get:

Probability of winning = 1 / (16 + 1)

Probability of winning = 1/17

In this case, the odds of 16:1 against winning correspond to a probability of 1/17, which represents the chance of winning the bet.

To learn more about probability click on,

https://brainly.com/question/29111123

#SPJ4

The triangles are similar. Find the value of x.

Answers

Since the triangles are similar, the value of x is equal to: C. 18 units.

What are the properties of similar triangles?

In Mathematics, two (2) triangles are said to be similar when the ratio of their corresponding side lengths are equal and their corresponding angles are congruent.

By applying the properties of similar triangles, we have the following ratio of corresponding side lengths;

AC/RS = AB/RT

By substituting the given side lengths into the above equation, we have the following:

x/24 = 24/32

By cross-multiplying, we have the following;

32x = 24(24)

32x = 576

x = 576/32

x = 18 units.

Read more on triangle here: brainly.com/question/22891315

#SPJ1

Missing information:

The question is incomplete and the complete question is shown in the attached picture.

The point on the graph represents Ann's location. She is using a metal detector on the beach to see what she can find. Each unit on the graph represents 2 feet. A pile of bottle caps is located at (4, -10). Find the length of the most direct path between Ann and the pile of bottle caps. Round to the nearest whole number.

Answers

Answer:

30 feet

Step-by-step explanation:

Coordinates of Ann: (-4,3)

Coordinates of bottle caps: (4,-10)

Distance from Ann to bottle caps can be found out using the distance formula:
[tex]x_2=4, x_1=-4\\y_2=-3,y_1=-10\\Distance=\sqrt{(x_{2}-x_{1})^2 + (y_{2}-y_{1})^2 } \\=\sqrt{(4-(-4))^2 + ((-10)-3)^2} \\=15.26\\15\text{ is the answer}[/tex]

Jason has a block of clay that is made up of two rectangular pieces of
different colors. Find the volume of the block of clay.
The measurements of the two clay blocks are:
6 cm
4 cm
3 cm
5 cm

Answers

Answer: 132 cubic centimeters

Step-by-step explanation:

To find the volume of the block of clay, we need to add the volumes of the two rectangular pieces.

The volume of a rectangular solid can be found by multiplying its length, width, and height. Let's call the first rectangular piece A and the second rectangular piece B. Then the dimensions of A are 6 cm (length), 4 cm (width), and 3 cm (height), and the dimensions of B are 5 cm (length), 4 cm (width), and 3 cm (height).

The volume of A is:

Volume of A = length x width x height = 6 cm x 4 cm x 3 cm = 72 cubic centimeters

The volume of B is:

Volume of B = length x width x height = 5 cm x 4 cm x 3 cm = 60 cubic centimeters

So the total volume of the block of clay is:

Volume of block = Volume of A + Volume of B = 72 cubic centimeters + 60 cubic centimeters = 132 cubic centimeters

Therefore, the volume of the block of clay is 132 cubic centimeters.

How to turn 0. 1212121212 into a simplified fraction

Answers

Answer:

  4/33

Step-by-step explanation:

You want to write 0.1212...(repeating) as a simplified fraction.

Repeating decimal

A repeating decimal beginning at the decimal point can be made into a fraction by expressing the repeating digits over an equal number of 9s.

Here, there are 2 repeating digits, so the basic fraction is ...

  12/99

This can be reduced by removing a factor of 3 from numerator and denominator:

  [tex]0.\overline{12}=\dfrac{12}{99}=\boxed{\dfrac{4}{33}}[/tex]

__

Additional comment

Formally, you can multiply any repeating decimal by 10 to the power of the number of repeating digits, then subtract the original number. This gives the numerator of the fraction. The denominator is that power of 10 less 1.

  0.1212... = (12.1212... - 0.1212...)/(10^2 -1) = 12/99

Doing this multiplication and subtraction also works for numbers where the repeating digits don't start at the decimal point. Finding a common factor with 99...9 may not be easy.

You can also approach this by writing the number as a continued fraction. The basic form is ...

  [tex]x=a+\cfrac{1}{b+\cfrac{1}{c+\cdots}}[/tex]

where 'a' is the integer part of the original number, and b, c, and so on are the integer parts of the inverse of the remaining fractional part. The attachment shows how this works for the fraction in the problem statement.

A calculator cannot actually represent a repeating decimal exactly, so error creeps in and may eventually become significant.

the stopping distance s of a car varies directly as the square of its speed v. if a car traveling at 40 mph requires 80 ft to stop, find the stopping

Answers

If a car traveling at 40 mph requires 80 feet to stop, the stopping distance S of a car varies directly as the square of its speed v and is equal to 180 feet.


Given, the stopping distance S of a car varies directly as the square of its speed v. So the relation can be represented as,

S ∝ v2

Here, the constant of proportionality is k.

S = kv2 ——— (1)

Given, when the speed v = 40 mph, stopping distance s = 80 feet.

Therefore, from equation (1), we have

80 = k × 402

k = 80/1600

k = 0.05

Hence, the relation between the stopping distance S and the speed v of the car can be given as

S = 0.05v2

To find the stopping distance S of the car at speed v = 60 mph, substitute v = 60 in the above equation.

S = 0.05 × 602

S = 0.05 × 3600

S = 180 feet

Therefore, the stopping distance of a car traveling at 60 mph would be 180 feet.


To know more about stopping distance refer here:

https://brainly.com/question/24254597#

#SPJ11

The supplement of an angle is 30 more than twice its complement. What is the measure of the
angle?

Answers

Answer: 30

180 - x = 180 - 2x + 30

x = 30

Answer:

The measure of the unknown angle is 30°.

Step-by-step explanation:

Let the measure of the unknown angle be x°.

Supplementary angles are two angles whose measures sum to 180°.

Complementary angles are two angles whose measures sum to 90°.

Therefore, the supplement of x° is (180 - x)°, and its complement is (90 - x)°.

Given that the supplement is 30° more than twice its complement:

(180 - x)° = 2(90 - x)° + 30°

To find the measure of the angle, solve the equation:

⇒ (180 - x)° = (180 - 2x)° + 30°

⇒ 180° - x° = 180° - 2x° + 30°

⇒ 180° - x° = 210° - 2x°

⇒ 180° - x° + 2x° = 210° - 2x° + 2x°

⇒ 180° + x° = 210°

⇒ 180° + x° - 180° = 210° - 180°

⇒ x° = 30°

Therefore, the measure of the unknown angle is 30°.

Out of 80 customers at an ice cream van, 48 had syrup, 28 had sprinkles and 16 had both
toppings on their ice cream. Use a Venn diagram to find the probability that a randomly
selected customer doesn't have either topping, given that they don't have sprinkles.

I know the answer is 20/52, I just can’t work out how to get to that answer…

Answers

Answer:

20/52 or simplified to 5/13

Step-by-step explanation:

The Venn Diagram is provided

Let
n(A) =  number of customers who had syrup
n(B) =  number of customers who had sprinkles

n(A and B) = number of customers who had both syrup and sprinkles = 16
This would be the number in the overlapping region

n(A or B) = number of customers who had either syrup or sprinkles or both
= n(A) + n(B) - n(A and B)
= 48 + 28 - 16
= 60

Therefore number of customers who had neither topping = 80 - 60 = 20

This number is indicated outside both circles but within the rectangle

The number of customers who had only syrup is given by set difference

= No. of customers who had syrup - No. of customers who had both
= n(A) - n(A and B)
= 48 - 16
= 32

This is the figure inside the left circle

Let's consider the statement: Customers who didn't have sprinkles

This would be customers who had only syrup(32) + customers who had neither topping(20)
= 32 + 20 = 52

Number of customers who did not have either topping = 20
P(selected customer doesn't have either topping, given that they don't have sprinkles)
= 20/52
= 5/13

you roll a dice with 6 sides what is the probability of....... write your answer as a fraction....roll a 5,roll a 6,roll a c or 4,roll an odd number,roll an even nuber,roll a number greater than 3?,roll an even number less that 5?,roll a multiple of 2(2,4,6),roll a factor of 6(6,4,2,1).

Answers

Roll a 5: There is only one way to roll a 5 out of six possible outcomes, so the probability of rolling a 5 is 1/6.

Roll a 6: Similarly, there is only one way to roll a 6 out of six possible outcomes, so the probability of rolling a 6 is 1/6.

Roll a c or 4: There are two ways to roll a 4 (rolling a 4 or rolling a 3) and one way to roll a 3, so there are three ways to roll a 4 or c out of six possible outcomes. Therefore, the probability of rolling a 4 or c is 3/6, simplifying it to 1/2.

Roll an odd number: There are three odd numbers (1, 3, 5) out of six possible outcomes, so the probability of rolling an odd number is 3/6, simplifying to 1/2.

Roll an even number: There are three even numbers (2, 4, 6) out of six possible outcomes, so the probability of rolling an exact number is 3/6 or 1/2.

Roll a number greater than 3: There are three numbers greater than 3 (4, 5, 6) out of six possible outcomes, so the probability of rolling a number greater than 3 is 3/6, which simplifies to 1/2.

Roll an even number less than 5: There is only one number less than 5 (2) out of six possible outcomes, so the probability of rolling an actual number less than 5 is 1/6.

Roll a multiple of 2 (2, 4, 6): There are three multiples of 2 out of six possible outcomes, so the probability of rolling a multiple of 2 is 3/6, simplifying to 1/2.

Roll a factor of 6 (1, 2, 3, 6): There are four factors of 6 out of six possible outcomes, so the probability of rolling a factor of 6 is 4/6, which simplifies to 2/3.

So the probabilities for each event expressed as fractions are:

Roll a 5: 1/6

Roll a 6: 1/6

Roll a 4 or c: 1/2

Roll an odd number: 1/2

Roll an even number: 1/2

Roll a number greater than 3: 1/2

Roll an even number less than 5: 1/6

Roll a multiple of 2: 1/2

Roll a factor of 6: 2/3

Proportions

Two plus x divided by twelve equals one dived by three. Solve for x.

Answers

Two plus x divided by twelve equals one divided by three

Case 1 :

Rewrite into numbers : 2 + x /12 = 1/3

-> x/12 = 1/3 - 2 = -5/3

-> x = -5/3 x 12 = -20

Case 2 :

Rewrite into numbers : (2 + x)/12 = 1/3

-> 2 + x = 1/3 x 12 = 4

-> x = 4 - 2 = 2

i dont know if you meant it the right way or the wrong way but ill just put them both

x=2

Step-by-step explanation:

(2+x)/12=1/3

3(2+x)=12

2+x=4

x=4-2

x=2

Other Questions
construct the confidence interval for the population standard deviation for the given values. Round your answers to one decimal place. n=5, s=4.4, c=0.95 jade almost got hit by a car at a street corner because she was too busy texting on her phone. from that day on, jade looks before she reaches the street corner. her change in behavior is a result of which of these can be used to represent octane? group of answer choices c8h18 ch3(ch2)6ch3 ch3ch2ch2ch2ch2ch2ch2ch3 all of these are correct what is the gene density of the human genome? calculate your answer in genes per mb (mega base pairs, or 1 million base pairs). ReadWorksA New Kind of Library - Comprehension Ques5. What is this passage mostly about?A. how technology and computers changed the New York Public LibraryB. how public libraries in cities across the United States functionC. how the New York Public Library has changed the way people use computersD. how crowdsourcing can help many people become involved in a projectI need this today :( all of the windings of three phase motors are always wired how? the clinician is assessing for the most common cause of increased neck size. which area would the clinician exam? ben wanted to know how many college students living in the dorms subscribed to a streaming service such as netflix, but he couldn't survey every one of them. ben decided that in order to give all students an equal chance of being selected to participate in his mini study, he would visit the dorms at dinnertime and ask them to choose numbers out of a hat. the ones with the number 7 were given a quick three-question survey. ben is demonstrating coachlight incorporated uses a periodic inventory system. the company purchased 200 units of inventory at $20.00 per unit and 300 units at $21.00 per unit. what is the weighted average unit cost for these purchases of inventory? (round your final answer to two decimal places.) over the past year, price inflation has been 10%, but the price of a used ford escort has fallen from $6,000 to $5,000. the real price of a ford escort has fallen by: Given (x 7)2 = 36, select the values of x. x = 13 x = 1 x = 29 x = 42 the organ dysfunction that has increased pressure in the abdominal cavity due to ischemia of abdominal tissues is called: Rose has made this scale drawing of her house. If her house actually 56 feet wide, then scale of her drawing is when people experience a loss of self-awareness as well as evaluation apprehension, they are in a state of According to the pecking-order theory proposed by Stewart Myers of MIT, which of the following are correct? I. For financing needs, firms prefer to first tap internal sources such as retained profits and excess cash. II. There is an inverse relationship between a firm's profit level and its debt level. III. Firms prefer to issue new equity rather than source external debt. IV. A firm's capital structure is dictated by its need for external financing. A sphere has a surface area of 60 square feet. Which choice is the best approximation of its radius? Use 3.14 to approximate pi.\ Tolosa sold 540 eggs if these are 35% of total eggs,then how many eggs are not sold? Jim worked 45 hours this week. He earns time and a half for overtime. He is paid $12.59 per/hour, how much will he earn this week? Rewrite each expression using the distributive property.then combine like terms if possible-8(x+4)+5(x+4) A tortoise is walking in the desert. It walks for 6.4 meters at a speed of 4 meters per minute. For how many minutes does it walk?